1
$\begingroup$

Consider a branching random walk given by the collection of i.i.d random variables $X(i_{0},\ldots,i_{t})$. Here $t \in \mathbb{N}$ and $i_{k} \in \{1,\ldots,n\}$ for any $k \in \mathbb{N}$. Each $X(i_{0},\ldots,i_{t})$ is Bernoulli, taking value 0 with probability $p$ and 1 with probability $1-p$. Consider the maximal displacement of the BRW at time $t$: $$M_{t} = \max_{(i_{0},\ldots,i_{t})}\sum_{k = 0}^{t} X(i_{0},\ldots,i_{k}).$$ I am interested in the asymptotic behaviour of $M_{t}/t$. Specifically:

(1) Is it correct that $\tfrac{M_{t}}{t} \to 1$ a.s. if $p < 1-\tfrac{1}{n}$? (My logic is to apply the textbook percolation result. In fact, it even seems to tell me more: a.s. there is a $t_{0} \in \mathbb{N}$ such that $M_{t} \geq t-t_{0}$ for all $t \geq t_{0}$.)

(2) Is it correct that the previous conclusion is still true if $p = 1-\tfrac{1}{n}$?

(3) What can I say if $1-\tfrac{1}{n} < p$?

$\endgroup$
5
  • 2
    $\begingroup$ I think there is a significant number of typos, please correct. For example, the sum in the RHS of the definition of $M_t$ does not have $t$ in it, so how do you take the indicated max? Also, this seems not to be a BRW. Finally, how do you couple the different r.v.'s when you change $n$ (in order to discuss a.s. behavior)? Depending on the answer, the claim could be true for and $p>0$, and $p>p_c$ with $p_c<1$, or just false. $\endgroup$ Commented Sep 29 at 21:56
  • $\begingroup$ @oferzeitouni: thanks for your comment! typo corrected. My use of the term "BRW" seems to agree with that in Bramson MD: Maximal displacement of branching Brownian motion. Communications on Pure and Applied Mathematics. 1978 Sep;31(5):531-81. As for the question of "coupling": I suppose we may think of all the increments defined on the same probability space, for instance the product space $\{0,1\}^{T}$ where $T$ is the complete $n$-ary tree. $\endgroup$ Commented Sep 30 at 11:06
  • $\begingroup$ Now that $n$ is not a parameter in the limit, the question is well defined. $\endgroup$ Commented Oct 1 at 1:01
  • 1
    $\begingroup$ Look at Theorem 1.3 of Zhan Shi's St Flour course (Springer LNM 2151). After obvious change of notation and sign change, you will see that if $n(1-p)\geq 1$ then (in his notation) the infimum of $\psi(t)/t$ is obtained at $t\to \infty$, which gives velocity=1, while otherwise the velocity is the infimum of $\log(np+n(1-p)e^t))/t$ over $t>0$. $\endgroup$ Commented Oct 1 at 1:52
  • $\begingroup$ @oferzeitouni: thanks! question answered. $\endgroup$ Commented Oct 5 at 14:00

1 Answer 1

1
$\begingroup$

Look at Theorem 1.3 of Zhan Shi's St Flour course (Springer LNM 2151). After obvious change of notation and sign change, you will see that if $n(1−p)≥1$ then (in his notation) the infimum of $ψ(t)/t$ is obtained at $t\to \infty$, which gives velocity=1, while otherwise the velocity is the infimum of $\log(np+n(1−p)e^t))/t$ over $t>0$.

$\endgroup$
1

You must log in to answer this question.

Start asking to get answers

Find the answer to your question by asking.

Ask question

Explore related questions

See similar questions with these tags.